4

Given $(\mathbb R^n,\mathcal B,\mu)$, where $\mu$ is a positive finite Radon measure, define the function $$ \mu^*(A) = \inf \left\{ \sum_{i=1}^n\mu(B_i) \,\,|\,\, A\subseteq \cup_i B_i, \,\, n \in \mathbb N \right\} $$ where $B_i$ are balls (both closed and open).

Take now $G:\mathbb R^n \to \mathbb R_+$ a continuous function with compact support $E$ and call $$ E_r = \{x\in \mathbb R^n | G(x)>r \}. $$ Given any $\varepsilon>0$, is it true that $$ \mu^*(E_\epsilon) \le \mu(E_0)? $$

mathworker21
  • 34,399
Exodd
  • 10,844
  • Isn't $E_r$ an open set and thus a countable union of balls, implying $\mu^*(E_r) = \mu(E_r)$, which is at most $\mu(E_0)$ for any $r > 0$? – mathworker21 Sep 11 '19 at 09:27
  • @mathworker21 in the definition of $\mu^*$ the sum is over a finite number of balls that cover $E_r$ – Exodd Sep 11 '19 at 09:31
  • Yes, but you have continuity. That is, if $E_r = \cup_{n=1}^\infty B_n$, then $\sum_{i=1}^n \mu(B_i) = \mu(\cup_{i=1}^n B_i) \to \mu(\cup_{i=1}^\infty B_i) = \sum_{i=1}^\infty \mu(B_i)$. I should have said in my first comment "countable union of pairwise disjoint closed balls". – mathworker21 Sep 11 '19 at 09:33
  • @mathworker21 first of all, is it true that an open set is the union of countable disjoint closed ball? But even if it is true, $\mu^*$ is not countable addictive or subaddictive.. – Exodd Sep 11 '19 at 09:38
  • Sorry, you're right, once again. Thanks a lot! – mathworker21 Sep 11 '19 at 09:51
  • May I ask why your question isn't: prove $\mu^*(E_\epsilon) \le \mu(E_0)$ for every $\epsilon > 0$? – mathworker21 Sep 11 '19 at 10:07
  • @mathworker21 yeah, $\mu^*$ is monotone. I'll edit – Exodd Sep 11 '19 at 11:20
  • Hi. Do you know what $\mu^*({x \in \mathbb{R}^2 : r < |x| < R})$ is? If it's strictly more than $\pi(R^2-r^2)$, then I have a counter-example. – mathworker21 Sep 12 '19 at 08:55
  • @mathworker21 Actually no, since I need the result for any $\mu$ finite Radon measure. Do you need that bound for every $r<R$ or only for specific ones? – Exodd Sep 12 '19 at 11:47
  • @mathworker21 $\mu$ is a finite measure, so you cannot choose Lebesgue... Or am I missing something? – Exodd Sep 12 '19 at 11:51
  • I think it's confusing to use the term "outer measure" for your object $\mu^$, since it is quite different from the usual notion of outer measure. For instance, if $\mu$ is Lebesgue measure on $[0,1]$, then in your notation we have $\mu^(\mathbb{Q}) = 1$. Your $\mu^*$ is more like an "outer content". – Nate Eldredge Oct 06 '19 at 02:09
  • @Exodd by the way, you're welcome for spending so much time on your question – mathworker21 Dec 12 '19 at 02:04

2 Answers2

3

Partial answer.

Here's a proof for $n=1$. We actually show that $E_\epsilon$ is the union of finitely many pairwise disjoint balls. Since $\{x : G(x) > 0\}$ is open, we may write it as $\{G > 0\} = \sqcup_{n=1}^\infty (a_n,b_n)$, a countable union of disjoint intervals (the proof is just to take maximal intervals in $\{G > 0\}$). Now, since $G$ is uniformly continuous (it has compact support), there is some $\delta > 0$ with $|x-y| < \delta \implies |G(x)-G(y)| < \epsilon$. Since $\{G > 0\}$ is bounded, $\infty > m(\{G > 0\}) = \sum_n m((a_n,b_n)) = \sum_n b_n-a_n$, where $m$ is the Lebesgue measure. Therefore, for all except $N < \infty$ intervals, $b_n-a_n < \delta$. If $b_n-a_n < \delta$, then since $G(a_n) = 0$, $G(x) < \epsilon$ for $x \in (a_n,b_n)$. We conclude that $E_\epsilon$ is the union of finitely many pairwise disjoint balls.

.

Here's a proof for $n=2,3$ for any measure $\mu$ that is absolutely continuous with respect to the Lebesgue measure. Throughout, $n$ refers to $2$ or $3$ and $|\cdot|$ refers to $\mu$.

.

Lemma: let $Q$ be a closed square in $\mathbb{R}^n$. Then, for any $\epsilon > 0$, there is a collection of open balls $\{B_j\}_j$ such that $\sum_{j=1}^\infty |B_j| \le |Q|+\epsilon$.

Proof: Start with $\{B_j'\}$ an Apollonian gasket in $\mathbb{R}^n$. (This is why we need $n \le 3$, I think). The set $E_0 := Q\setminus \cup_j B_j'$ has measure $0$, so let $U$ be an open set containing $E_0$ with $|U| \le \epsilon$. By Vitali's covering Lemma, there is a collection $\{B_j''\}_j$ covering $U$ with $\sum_j |B_j''| \le C_n|U| \le C_n\epsilon$. So, $\{B_j\}_j := \{B_j'\}_j \cup \{B_j''\}_j$ covers $E_0$ and has $\sum_j |B_j| \le |Q|+C_n\epsilon$.

.

Main Proof: For $r > 0$, we show $\mu^*(E_r) \le \mu(E_r)$. Take $\epsilon > 0$. Take $0 < r'' < r$ so that $|E_{r''}| < |E_r|+\epsilon$. Take any $r' \in (r'',r)$. Since $E_{r''}$ is open, there exist a collection of closed cubes $\{Q_j\}_j$ that are disjoint except for overlapping boundaries and $E_{r''} = \cup_j Q_j$. By compactness (and looking at the interior of the $Q_j$'s), there is a finite subcollection $Q_1,\dots,Q_N$ whose union covers $E_{r'}$. By the Lemma, for each $1 \le i \le N$, there is $\{B_j^{(i)}\}_j$ with $Q_i \subseteq \cup_j B_j^{(i)}$ and $\sum_{j=1}^\infty |B_j^{(i)}| \le |Q_i|+\frac{\epsilon}{N}$. Let $\{B_j\}_j$ denote $\{B_j^{(i)}\}_{i,j}$. Then, by compactness, there are $B_1,\dots,B_M$ covering $\overline{E_r}$. We therefore have $\mu^*(E_r) \le \mu^*(\overline{E_r}) \le \sum_{j=1}^M |B_j| \le \sum_{i=1}^N \sum_j |B_j^{(i)}| \le \sum_{i=1}^N |Q_i|+\epsilon \le |E_{r''}|+\epsilon \le |E_r|+2\epsilon$.

.

Remark: absolute continuity of $\mu$ was used in not caring about the overlapping boundaries.

mathworker21
  • 34,399
  • I just realized that our argument in chat actually works for the original problem with a little modification.. – Exodd Sep 12 '19 at 17:53
  • @Exodd I thought this was our argument in chat – mathworker21 Sep 13 '19 at 00:07
  • I'm saying I can make it work also for all finite measures – Exodd Sep 13 '19 at 07:16
  • @Exodd post it as an answer – mathworker21 Sep 14 '19 at 01:23
  • wait. It is false that open $\implies$ union of disjoint closed balls.. – Exodd Sep 14 '19 at 07:37
  • https://math.stackexchange.com/questions/1964888/can-open-set-in-mathbbrn-be-written-as-the-union-of-countable-disjoint-clo – Exodd Sep 14 '19 at 07:52
  • I want to think about it more first. I'll post any thoughts I have, and hopefully vice-versa – mathworker21 Sep 18 '19 at 08:53
  • Actually, you can prove it using Lemma 3.9 and Theorem 3.10 of http://poisson.phc.dm.unipi.it/~fpmaiale/notes/TGM.pdf but I don't know if I can trust these results – Exodd Sep 18 '19 at 10:45
  • @Exodd I figured out how to cover a square with balls nearly optimally. See the updated answer. I think the general proof will work as described in the notes you linked to right above. May I ask why you doubt them? – mathworker21 Oct 02 '19 at 12:27
  • I don’t see why in the first paragraph we can conclude that $E_\epsilon$ is a union of finitely many pairwise disjoint intervals, because some intervals $(a_n,b_n)$ constituting ${G>0}$ a priori can contain infinitely many intervals constituting $E_\epsilon$. In fact, $E_\epsilon$ can be a union of infinitely many pairwise disjoint intervals. For instance, for $\epsilon=1$ and put $G(x)=1+x\sin\tfrac{2\pi}x$ for $x\in [0,1]$, $G(x)=0$ for $x\in (-\infty,-1]\cup [2,\infty)$, and $G$ is linear at the segments $[-1,0]$ and $[1,2]$. – Alex Ravsky Oct 04 '19 at 09:01
  • @Exodd There are Wikipedia articles on Besicovitch covering theorem and Vitali covering lemma. Can you prove the requred claim using these or similar results? If $\mu(E)=0$ then outer regularity of $\mu$ and Besicovitch covering theorem imply $\mu^*(E)=0$. – Alex Ravsky Oct 04 '19 at 10:02
  • @AlexRavsky My first paragraph I think spells everything out. Can you tell me what you specifically object to? Like, which claim or implication do you object to? – mathworker21 Oct 04 '19 at 10:34
  • How we conclude that $E_\epsilon$ is a union of finitely many pairwise disjoint balls? – Alex Ravsky Oct 04 '19 at 10:46
  • A question for an other part of the proof. Why there is a finite subcollection $Q_1,\dots,Q_N$ whose union covers $E_{r'}$? By continuity of $G$, $\overline{E_{r'}}\subset E_{r’’}$, but why $\overline{E_{r'}}\subset\bigcup\operatorname{int} Q_i$? – Alex Ravsky Oct 04 '19 at 10:47
  • 1
    @AlexRavsky We tried to use Besicovitch and the regularity, but it does not seem so obvious. If you have a complete solution, then post an answer – Exodd Oct 04 '19 at 10:56
  • 1
    @mathworker21 I doubt the notes because they are.. notes. And something in the proofs doesn't strike me as totally exact. – Exodd Oct 04 '19 at 10:57
  • @AlexRavsky Thanks for your questions, but I don't see how you can be objecting at that part, sorry. I said $G(x) < \epsilon$ for $x \in (a_n,b_n)$ if $b_n-a_n < \delta$. And I said only finitely many $n$ have $b_n-a_n > \delta$. So ${G > \epsilon}$ is contained within the union of those finitely many intervals. – mathworker21 Oct 04 '19 at 10:57
  • @AlexRavsky In regards to that other part of the proof, thanks. Can we agreed to, instead of what was done in the proof, make the countably many $Q_i$'s a bit larger (each $\epsilon/2^i$ larger in measure) and open, and then apply compactness? – mathworker21 Oct 04 '19 at 11:02
  • I don't think $r''$ is needed. We can just use $r'$, stick with countably many $Q_i$'s, and do an $\epsilon/2^n$ instead of $\epsilon/N$. I was being stupid -- there was no need to get finitely many $Q_i$'s. I don't really feel like changing it, especially if Exodd needs a complete solution. In all honesty, I trust the notes and think someone can just read through them to see that the proofs are right. – mathworker21 Oct 04 '19 at 11:04
  • @Exodd I have a solution only for the case $μ(E)=0$. The general case looks hard so I think it is suitable for MathOverflow. – Alex Ravsky Oct 04 '19 at 11:05
  • 1
    @AlexRavsky we already have a solution assuming those notes are correct, which they nearly certainly are! all you have to do is cover an arbitary open set well by balls – mathworker21 Oct 04 '19 at 11:09
  • I agree that ${G>\epsilon}$ is contained within the union of those finitely many intervals. But these intervals constitute ${G>0}$, not ${G>\epsilon}$. So a priori it can happen that an interval $(a_n,b_n)$ constituting ${G>0}$ contains infinitely many disjoint intervals, constituting ${G>\epsilon}$. Also I recall that above I provided an example of $E_1$ which is a union of infinitely many pairwise disjoint intervals. – Alex Ravsky Oct 04 '19 at 11:15
  • @AlexRavsky I said "$G(x) < \epsilon$ if $x \in (a_n,b_n)$". it seems like you disagree with this. correct? – mathworker21 Oct 04 '19 at 11:20
  • No, I agree with this. – Alex Ravsky Oct 04 '19 at 11:21
  • Sigh. I'm an idiot. I'm really sorry. I get what you're saying now. (Did you edit your comment to make 'contained' italicized? that makes your point very clear, and I don't know how I didn't see that before....) Anyways, I guess ${G > \epsilon}$ is the union of infinitely many disjoint intervals, so we can approximate it's measure well via finitely many. I don't know why I talked about ${G > 0}$ to begin with. I don't want to think about this right now. – mathworker21 Oct 04 '19 at 11:32
  • OK. Concerning the other part of the proof. In the first comment, if we shrink a finite cover consisting of bit larger $Q’$ back, it can happen that it does not cover $\overline{E_{r'}}$. But the idea in the next comment looks OK. By the way, in the lemma formulation and its last sentence is missed that ${B_j}$ covers $Q$. – Alex Ravsky Oct 04 '19 at 11:39
  • I have downvoted this answer since it appears to use absolute continuity of $\mu$ with respect to Lebesgue measure in an essential way, whereas there is no reason that a positive finite Radon measure has this property. For instance, Dirac measure satisfies the hypotheses of the question but is not absolutely continuous, and consequently there has to be some argument for dealing with the overlaps. – pre-kidney Oct 05 '19 at 23:49
  • @pre-kidney I specifically said "for any measure that is absolutely continuous with respect to the Lebesgue measure", so obviously I know not every measure is absolutely continuous with respect to the Lebesgue measure. In any event, I'm not really sure it is that essential. Can't we perturb the cubes in the answer slightly so that they avoid the support of the part of $\mu$ that is absolutely continuous with respect to the Lebesgue measure? Also, I'm confused why you wouldn't also downvote because I didn't do $n > 3$. Also also, I don't think it's common to downvote partial answers – mathworker21 Oct 06 '19 at 00:10
  • I have removed my downvote with the understanding that you will make it clearer (at the start of your answer) that it is partial, and needs a significant new idea to approach the full problem. By the way, you will need to perturb spheres, not cubes, and I believe this is the most difficult part of the argument. In fact, I am not convinced that there is a positive answer to the question due to the possibility of constructing a very complicated Radon measure with support in a fractal set that can't be "perturbed" around in some way. – pre-kidney Oct 06 '19 at 00:18
  • @pre-kidney I made it clear that my answer is only partial. I don't like using phrases like "needs a significant new idea", because we don't know that ("need" is a strong word). I think perturbing cubes is what is relevant, not spheres. Also, a positive answer does follow from Lemma 3.9 / Theorem 3.10 in the set of notes mentioned in a comment above, and I do trust those notes. It is pretty weird that everyone (including me) has been lazy about checking those notes. – mathworker21 Oct 06 '19 at 00:22
  • Lemma 3.9 in those notes is incorrect (it has a missing / unstated assumption), here is a counterexample. Let $\mu$ be the Dirac measure at $0$, let $E_0={0}$, and let $\epsilon=\tfrac12$. Then there does not exist any cover of $E_0$ with sum of $\mu$-measure less than $\epsilon$ (because at least one set in the cover must contain $0$, pushing its $\mu$-mass up to $1$). I believe the lemma is intended to apply to measures $\mu$ which are absolutely continuous with respect to Lebesgue measure, in which case it appears to be correct. – pre-kidney Oct 06 '19 at 00:28
  • @pre-kidney $E_0$ is not a null-set. It has measure $1$? – mathworker21 Oct 06 '19 at 00:32
  • It is a Borel-null set – pre-kidney Oct 06 '19 at 00:33
  • @pre-kidney I don't know what that means. Borel is a sigma-algebra, not a measure. In any event, I want it to mean $\mu(E_0) = 0$. If the lemma means that and if the lemma is true, then if theorem 3.10 is also true, we have a proof for an affirmative answer to OP's question. – mathworker21 Oct 06 '19 at 00:34
  • @pre-kidney you're definitely misinterpreting. Look how the proof of Lemma 3.9 starts off. – mathworker21 Oct 06 '19 at 00:35
  • I agree with you the lemma appears to be correct if it is stated in a sufficiently precise way (i.e. $\mu$ is a locally finite Borel measure and $\mu(E_0)=0$) but then it appears other parts of the argument fall apart if you require $\mu$-null instead of Borel-null. I suggest you quote the precise versions of any results you wish to use from those third party notes in your answer... – pre-kidney Oct 06 '19 at 00:39
  • @pre-kidney $\mu$ is locally finite Borel measure and $\mu(E_0) = 0$. [I think OP is assuming $\mu$ is Borel]. Also, I didn't use any of this in my answer. I'm saying this stuff will yield an answer if the relevant things are true – mathworker21 Oct 06 '19 at 00:42
  • @pre-kidney any Radon measure is Borel by definition (https://en.wikipedia.org/wiki/Radon_measure) and in the notes, they always work with Borel measures (it is stated in pag. 9) – Exodd Oct 06 '19 at 08:25
  • @Exodd thanks for the clarification, I found the notes confusing to follow but the comments left by you and mathworker21 were enough to point me in the right direction of the Besicovitch covering theorem, which I have applied to give a complete and detailed proof. – pre-kidney Oct 06 '19 at 09:47
1

Yes, it is true, and to prove it I will use the following proposition.

Proposition. Let $\mu$ be any positive outer-regular Borel measure on $\mathbb R^n$, and let $K$ be any compact subset of $\mathbb R^n$. Then $\mu(K)= \inf \sum_{i=1}^k \mu(B_i)$, where the infimum is taken over all finite open covers $\lbrace B_1,\ldots,B_k\rbrace$ of $K$ by open balls $B_i$.

Before presenting the (somewhat lengthy) proof of the proposition, I will first show how it resolves the question. Consider the set $K=G^{-1}\bigl([\epsilon,\infty)\bigr)$, which is closed (by continuity of $G$) and compact (since it is a subset of the support of $G$, which was assumed to be compact). Since Radon measures are outer-regular and Borel, the proposition applies, yielding that $\mu^\star(E_\epsilon)\leq \mu(K)$. Since $K\subseteq E_0$, the bound $\mu^\star(E_\epsilon)\leq \mu(E_0)$ follows.

Proof of the proposition. Obviously $\mu(K)\leq \mu(B_1\cup\cdots\cup B_k)\leq \sum_{i=1}^k \mu(B_i)$ by subbaditivity, so $\mu(K)\leq \inf\sum_{i=1}^k \mu(B_i)$.

To establish the other inequality, we apply the following result:

Besicovitch Covering Theorem. Let $A$ be any subset of $\mathbb R^d$ and let $r\colon A\to\mathbb R_+$ be any function. Then there exists an integer $m\leq 5^d$ and disjoint subsets $A_1,\ldots,A_m$ of $A$ with the following properties:

  1. For all $1\leq i\leq m$, the collection of open balls centered at $x\in A_i$ with radius $f(x)$ is mutually disjoint.

  2. The union of all balls appearing in (1) contains $A$.

Fix $\epsilon>0$. If $\mu(K)=\infty$ there is nothing to prove, so suppose that $\mu(K)<\infty$. We will iteratively construct a cover of $K$ using open balls $B_i$ that are nearly disjoint. To start the iteration, set $K_0=K$. By outer regularity of $\mu$, there exists an open set $U_0$ containing $K_0$ such that $\mu(U_0)\leq(1+\epsilon)\mu(K_0)$. Cover each element of $K$ with a centered open ball contained in $U$. Apply the Besicovitch Covering Theorem to this cover. A disjoint subfamily with maximal $\mu$-measure will have measure at least $5^{-d}\mu(U_0)$. Since $\mu(K)<\infty$, there exists a finite disjoint subfamily - call it $C_0$ - with $\mu(C_0)\geq 6^{-d}\mu(U_0)$.

Given $(K_i,U_i,C_i)_{i=0}^n$ we continue the iteration by setting $K_{n+1}=K_n\setminus C_n$ and applying outer regularity to $K_{n+1}$ to find an open set $U_{n+1}$ containing $K_{n+1}$ with $\mu(U_{n+1}) \leq (1+\epsilon)\mu(K_{n+1})$. In fact, since $K_{n+1}\subseteq U_n$ we can further choose $U_{n+1}$ to be a subset of $U_n$ (by intersecting with $U_n$ if necessary). Cover $K_{n+1}$ with open balls in $U_{n+1}$ and apply Besicovitch to obtain a disjoint finite subcover $C_{n+1}$ with $\mu(C_{n+1})\geq 6^{-d}\mu(U_{n+1})$. These inequalities imply that $$ \mu(K_{n+1})\leq \mu(U_n)-\mu(C_n)\leq (1-6^{-d})\mu(U_n)\leq (1-7^{-d})\mu(K_n), $$ provided that $(1+\epsilon)(1-6^{-d})\leq (1-7^{-d})$ - which we can ensure by taking $\epsilon$ sufficiently small. Thus, we find that $\mu(K_n)$ decreases exponentially in $n$, and in fact $$ \sum_{n=0}^{\infty}\mu(K_n)\leq 7^d\mu(K). $$

Now it is time to bound the overlap between the $C_i$. Observe that $$ C_n\cap (C_0\cup\ldots\cup C_{n-1})\subseteq U_n\setminus K_n, $$ from which it follows by induction that $$ \mu(C_0)+\cdots+\mu(C_n)\leq\mu(C_0\cup \cdots\cup C_n)+\mu(U_1\setminus K_1)+\cdots+\mu(U_n\setminus K_n), $$ and we can bound the right side by $\mu(U_0)+\epsilon\sum_n \mu(K_n)$ (using the monotonicity $U_i\subseteq U_0$ for all $i$) to obtain an upper bound of $$ \mu(C_0)+\cdots+\mu(C_n)\leq (1+\epsilon+\epsilon 7^d)\mu(K).\qquad (\star) $$

We stop the iteration when $n$ becomes large enough so that $(1-7^{-d})^n\leq \epsilon$, in which case $\mu(K_n)\leq \epsilon\mu(K)$ and $\mu(U_n)\leq \epsilon(1+\epsilon)\mu(K)$. Rather than taking $C_{n+1}$ to be a disjoint subfamily as before, we allow overlaps in this final step. First, apply Besicovitch to the balls in $U_{n+1}$ covering $K_{n+1}$ to obtain $m\leq 5^d$ subfamilies whose union still covers $K_{n+1}$. Then apply compactness of $K_{n+1}$ to extract a finite subcover $C_{n+1}'$. Observe that even though this finite subcover is not disjoint, thanks to Besicovitch we still have a weak bound of $$ \sum_{B\in C_{n+1}'}\mu(B)\leq 5^d\mu(U_{n+1}).\qquad (\star\star) $$

Finally, we form the cover $C_0\cup\cdots\cup C_n\cup C_{n+1}'$ of $K$, which we now reindex in terms of the underlying open balls as $B_1\cup\cdots\cup B_k$. Combining $(\star)$ and $(\star\star)$ then yields $$ \sum_{i=1}^k\mu(B_i)\leq (1+\epsilon+\epsilon 7^d+5^d\epsilon(1+\epsilon))\mu(K). $$ Taking $\epsilon$ to $0$ yields the inequality $$ \mu(K)\geq \inf\sum_{i=1}^k \mu(B_i), $$ completing the proof of the proposition. $\square$

pre-kidney
  • 30,223
  • you still miss some steps, like why there is a finite number of balls in $C_i$ and it seems you are assuming $U_{i+1}\subseteq U_i$ – Exodd Oct 06 '19 at 10:34
  • Those steps are not missing, I suggest you read the proof a little more carefully. The finite number of balls in $C_i$ is addressed, albeit briefly (if you need it spelled out in more detail, consider finite exhaustions of the entire subfamily; in the limit, the subfamily attains the $5^{-d}$ bound, which implies there exists a finite stopping point at which a weaker bound (I chose $6^{-d}$ in the proof) is attained). Regarding your second concern, can you point out where you believe $U_{i+1}\subseteq U_i$ is being used? By the way, finding typos helps, but clearly the idea works :) – pre-kidney Oct 06 '19 at 10:45
  • I see the finitness for $C_i$. You use the containment when you bound $\mu(C_0\cup \dots\cup C_n)$ with $\mu(U_0)$ – Exodd Oct 06 '19 at 10:48
  • Yes, you're correct, although it can be easily patched since the spillover has exponentially decaying measure (just like the second term). I will fix it. – pre-kidney Oct 06 '19 at 10:51
  • or you can simply say "since $K_i$ are decreasing, we can find $U_i$ decreasing as well" – Exodd Oct 06 '19 at 10:59
  • Yes, that is true as well - and was the route I ended up choosing when patching the answer. – pre-kidney Oct 06 '19 at 11:01
  • btw, you could also have taken $C_i$ infinite, and then extract a finite covering at the very end since $K$ is compact – Exodd Oct 06 '19 at 11:10